CARDIOLOGY Flashcards

1
Q

Which of the following extremity vein is considered part of the superficial venous system?

a. Small saphenous vein
b. Superficial femoral vein
c. Anterior tibial vein
d. Posterior tibial vein

A

Small saphenous vein

How well did you know this?
1
Not at all
2
3
4
5
Perfectly
2
Q

Pulmonary hypertension is best defined in mmHg as:

a. Mean PAP >20
b. Mean PAP >22
c. Mean PAP >24
d. Mean PAP >26

A

b. Mean PAP >22

How well did you know this?
1
Not at all
2
3
4
5
Perfectly
3
Q

This is a class Ia antiarrhythmic that has also an additional property of alpha-adrenergic blockade:

a. Quinidine
b. Procainamide
c. Lidocaine
d. Amiodarone

A

a. Quinidine

How well did you know this?
1
Not at all
2
3
4
5
Perfectly
4
Q

Which anatomical site determines most of the peripheral vascular resistance?

a. Medium-sized arteries and arterioles
b. Small and medium-sized arteries
c. Small arteries and arterioles
d. Arterioles and venoules

A

c. Small arteries and arterioles

How well did you know this?
1
Not at all
2
3
4
5
Perfectly
5
Q

In which of the following situations is CABG the preferred revascularization strategy?

a. Significant right main coronary artery involvement
b. Two-vessel CAD with significant left anterior descending involvement
c. Depressed left ventricular function (EF <50%)
d. Heavy cigarette smokers

A

c. Depressed left ventricular function (EF <50%)

How well did you know this?
1
Not at all
2
3
4
5
Perfectly
6
Q

A 50-year old male who was recently diagnosed to have hypertension asks regarding lifestyle modifications that effectively lower blood pressure. He is also a known diabetic, a chronic smoker and alcoholic. What is best to recommend for this case?

a. Maintain a BMI <23 kg/m2
b. Reduce total sodium intake to <5g/day
c. Regular aerobic activity for >30mins minimum of 3x a week
d. Reduce alcohol intake to 2 or ess drinks per day

A

d. Reduce alcohol intake to 2 or less drinks per day

How well did you know this?
1
Not at all
2
3
4
5
Perfectly
7
Q

At this level of coronary artery stenosis, blood flow at rest may be reduced and further decreases may produce ischemia at rest or with little effort:

a. 50%
b. 60%
c. 70%
d. 80%

A

d. 80%

How well did you know this?
1
Not at all
2
3
4
5
Perfectly
8
Q

In which of the following situations is coronary angiography NOT indicated to evaluate angina pectoris?

a. Severely symptomatic patients despite optimal medical therapy and are being considered for revascularization
b. Patients with known or suspected angina pectoris who have survived cardiac arrest
c. Patients with angina symptoms and nondiagnostic stress test but are experiencing psychological stress regarding the definitive diagnosis
d. None of the above

A

d. None of the above

How well did you know this?
1
Not at all
2
3
4
5
Perfectly
9
Q

Ramon, a newborn baby, was immediately referred to the cardiology section for sudden dyspnea and cyanosis. The 12 L ECG read ST depression at V1-V4 and cardiac enzymes were elevated. Which of the following can explain the cause of the infarction?

a. Ostial narrowing due to aortitis
b. Origin of LAD from pulmonary artery
c. Severe LVH due to aortic stenosis
d. Severe atherosclerosis brought by familial hypercholesterolemia

A

b. Origin of LAD from pulmonary artery

How well did you know this?
1
Not at all
2
3
4
5
Perfectly
10
Q

Robert was doing his brisk walking when he suddenly felt severe pain at his left chest. He suddenly placed his hand over his sternum, clutching it with a clenched fist because of discomfort. What is the medical term when Robert clutched his sternum because of pain?

a. Waterhammer sign
b. Osler’s sign
c. Levine’s sign
d. Turner’s sign

A

c. Levine’s sign

How well did you know this?
1
Not at all
2
3
4
5
Perfectly
11
Q

A 55-year old man being treated for chronic stable angina pectoris consulted for worsening exertional dyspnea. His medications include a beta blocker, an ACE inhibitor and as needed furosemide for dyspnea symptoms. Transthoracic 2D echo revealed LVH, mild mitral regurgitation and an EF of 38%. What is the most appropriate next step?

a. Increase furosemide to a regular dose to relieve the dyspnea
b. Add an aldosterone-antagonist
c. Add a nitrate
d. Refer to coronary angiography

A

d. Refer to coronary angiography

How well did you know this?
1
Not at all
2
3
4
5
Perfectly
12
Q

A 74 year-old woman, with hypertension for 15 years and recently diagnosed with type 2 diabetes mellitus consults for a 2-week history of increasing fatigability and bipedal edema. She is maintained on Telmisartan, Hydrochlorothiazide, Pioglitazone and Rosuvastatin. A complete blood count, creatinine and electrolyte panel done two days ago are all within normal. Transthoracic 2d-echocardiogram showed concentric LVH and ejection fraction of 40%. Which medical strategy can be used to optimize heart failure control and potentially decrease mortality?

a. Add a long-acting nitrate
b. Add Atenolol
c. Shift Telmisartan to Olmesartan
d. Add Bisoprolol

A

d. Add Bisoprolol

How well did you know this?
1
Not at all
2
3
4
5
Perfectly
13
Q

Which of the following may cause transient elevations of cardiac troponins in a patient without a clear history of myocardial ischemia?

a. Liver failure
b. Bradycardia
c. Hypothyroidism
d. Aortic aneurysm

A

c. Hypothyroidism

How well did you know this?
1
Not at all
2
3
4
5
Perfectly
14
Q

A 54 year old diabetic woman is in the emergency room for acute onset crushing chest pain. ECG showed T-wave inversions in V2-V6 and high sensitivity troponin T taken 6 hours after the onset of chest pain was 10x elevated. She was pale and diaphoretic, BP of 100/80mmHg, HR of 115, RR of 30, with engorged neck veins and crackles all over both lung fields and ongoing chest pain. She was immediately hooked to non-invasive positive pressure ventilation wherein her tachypnea lessened.Which of the following medications are indicated and can be safely given?

a. Metoprolol 50mg PO every 6 hours
b. Verapamil 5 mg IV every 6 hours
c. Morphine 2 mg IV every 30 mins and as needed
d. Rivaroxaban 20 mg BID

A

c. Morphine 2 mg IV every 30 mins and as needed

How well did you know this?
1
Not at all
2
3
4
5
Perfectly
15
Q

Which of the following patient conditions is an absolute contraindication for fibrinolysis in STEMI?

a. Traumatic subarachnoid hemorrhage during childhood
b. 2 cm infrarenal aortic aneurysm
c. Third trimester pregnancy
d. Heavy menstruation

A

a. Traumatic subarachnoid hemorrhage during childhood

How well did you know this?
1
Not at all
2
3
4
5
Perfectly
16
Q

According to the Philippine Guidelines in the treatment of dyslipidemia what is appropriate to advise a 55 year old male executive, previously asymptomatic individual came in to your clinic for routine executive checkup, non-hypertensive, 27 pack-year smoker with the following laboratory results: Hba1c- 7.5%, Total cholesterol- 130 mg/dL, LDL cholesterol- 95 mg/dL, HDL cholesterol- 35 mg/dL, Triglycerides- 140 mg/dL, Creatinine- 0.80 umol/L?

a. Low fat, low cholesterol diet
b. Diet high in fruits and vegetables
c. Smoking cessation
d. All of the above

A

d. All of the above

How well did you know this?
1
Not at all
2
3
4
5
Perfectly
17
Q

In which of the following situations is CABG the preferred revascularization strategy according to the Philippine Heart Association Guidelines?

a. Significant right main coronary artery involvement
b. Two-vessel CAD with significant left anterior descending involvement
c. Depressed left ventricular function (EF <50%)
d. Heavy cigarette smokers

A

c. Depressed left ventricular function (EF <50%)

How well did you know this?
1
Not at all
2
3
4
5
Perfectly
18
Q

Following an episode of myocardial infarction, the following are recommended medications by the Philippine Heart Association:

a. Aspirin for 12 months in patients treated with PCI and drug eluting stents
b. Clopidogrel for 12 months in patients treated with PCI and drug eluting stents
c. Prasugrel for 12 months in patients treated with medical management alone
d. Ticagrelor for 12 months in patients treated with PCI and bare metal stents

A

b. Clopidogrel for 12 months in patients treated with PCI and drug eluting stents

How well did you know this?
1
Not at all
2
3
4
5
Perfectly
19
Q

Which of the following is not included in the GRACE risk model to assess risk of death and MI in patients with NSTE-ACS and UA:

a. Age
b. Serum creatinine
c. History of heart failure
d. Diabetes

A

d. Diabetes

How well did you know this?
1
Not at all
2
3
4
5
Perfectly
20
Q

Which of the following is part of the criteria for metabolic syndrome under the Harmonizing Definition Criteria for Metabolic Syndrome?

a. Fasting triglyceride level of >180 mg/dL or medication
b. HDL cholesterol level <50 mg/dL for men
c. Fasting plasma glucose level of >100 mg/dL
d. Blood pressure of >130 mmHg systolic

A

d. Blood pressure of >130 mmHg systolic

How well did you know this?
1
Not at all
2
3
4
5
Perfectly
21
Q

The following is/are indication/s for emergent cardiac surgery in patients with infective endocarditis:

a. Vegetation obstructing valve tract
b. Septal perforation
c. Rupture into pericardial sac
d. All of the above

A

d. All of the above

How well did you know this?
1
Not at all
2
3
4
5
Perfectly
22
Q

What is the most widely accepted theory regarding the pathogenesis of rheumatic fever?

a. Delayed type hypersensitivity
b. Antigen-antibody deposition
c. Direct streptococcal invasion to heart valves
d. Molecular mimicry

A

d. Molecular mimicry

How well did you know this?
1
Not at all
2
3
4
5
Perfectly
23
Q

An 18-year old female who had a bout of acute rheumatic fever 5 years ago came in for follow-up. She is asymptomatic and her surveillance 2D echocardiogram showed moderate mitral regurgitation, stable since her initial episode. What is the most appropriate recommendation for secondary prevention of Acute Rheumatic Fever in this case?

a. Penicillin G 1.2 Million units IM every 3 weeks for at least 10 years or until she reaches 40 years of age
b. Penicillin G 1.2 Million units IM every 3 weeks for at least 5 years or until she reaches 21 years of age
c. Penicillin V 250 mg once a day for at least 10 years or until she reaches 40 years of age
d. Penicillin V 250 mg once a day for at least 5 years or until she reaches 21 years of age

A

a. Penicillin G 1.2 Million units IM every 3 weeks for at least 10 years or until she reaches 40 years of age

How well did you know this?
1
Not at all
2
3
4
5
Perfectly
24
Q

Which of the following statements is true about physical examination findings in aortic stenosis:

a. Atrial fibrillation often occurs early in the disease course, and may signal mitral valve involvement
b. Pulse pressure widening and systolic blood pressure decline may happen in later stages of aortic stenosis
c. There is often an ejection, mid-systolic murmur that begins shortly after S1
d. The carotid artery rises sharply, thus the classic pulsus parvus et tardus

A

c. There is often an ejection, mid-systolic murmur that begins shortly after S1

How well did you know this?
1
Not at all
2
3
4
5
Perfectly
25
Q

The following classic murmurs of aortic regurgitation is matched correctly with the description:

a. Corrigan’s pulse: a rapidly rising pulse and collapses suddenly as arterial pressure falls rapidly during late systole and diastole
b. Quincke’s pulse: paradoxical nail pallor when pressure is applied to the tip of the finger
c. Traube’s sign: to-and-fro murmur audible if the femoral artery is compressed lightly by a stethoscope
d. Duroziez’s sign: booming “pistol shot” sound heard over the femoral arteries

A

a. Corrigan’s pulse: a rapidly rising pulse and collapses suddenly as arterial pressure falls rapidly during late systole and diastole

How well did you know this?
1
Not at all
2
3
4
5
Perfectly
26
Q

Which of the following statements correctly describes mitral valve prolapse?

a. In MVP, the anterior leaflet is more often affected than the posterior leaflet
b. Auscultation findings may reveal an early to mid non-ejection systolic click that is generated by sudden tensing of the slack, elongated chordae tendinae
c. The murmur of MVP occur earlier during standing and Valsalva maneuver
d. Squatting and isometric exercises increase the MVP murmur

A

c. The murmur of MVP occur earlier during standing and Valsalva maneuver

How well did you know this?
1
Not at all
2
3
4
5
Perfectly
27
Q

A 64-year old female consulted for a non-healing wound in her left leg. She is non-diabetic and non-hypertensive and lipid profile is unremarkable. On physical examination, there is a dried and healed venous ulcer in the dorsal aspect of the left foot, lichenification in the left leg. Leg duplex ultrasound with doppler showed chronic venous incompetence in the left great and lesser saphenous veins. What is the recommended management for this case?

a. Prescribe diosmin+hesperidin to lessen venous incompetence
b. Prescribe low molecular weight heparin
c. Prescribe elastic stockings and stretch bandages
d. Refer to surgeon for interventional procedures

A

d. Refer to surgeon for interventional procedures

How well did you know this?
1
Not at all
2
3
4
5
Perfectly
28
Q

A 54 year old woman complained of progressive fatigue and dyspnea. A year ago, she also complained of mouth and eye drying, and difficulty swallowing and swallowing. She denies fever, weight loss and other constitutional symptoms. On physical examination, she is awake, and not in distress. She has limited mouth opening and cutaneous thickening at the dorsal aspects of the hands. Vital signs are normal, there’s a holosystolic murmur at the 5th ICS left parasternal border. 2D echo which showed markedly increased PAP (45 mmHg). Under what group classification of pulmonary hypertension does the patient belong? (based on the WHO Classification)

a. Group I
b. Group II
c. Group III
d. Group IV

A

a. Group I

How well did you know this?
1
Not at all
2
3
4
5
Perfectly
29
Q

This abnormality can also present with an ST Segment elevation and can be confused as a myocardial infarction:

a. Hypocalcemia
b. Hypokalemia
c. Hypothermia
d. Hypothyroidism

A

c. Hypothermia

How well did you know this?
1
Not at all
2
3
4
5
Perfectly
30
Q

Infiltration of abnormal substances between myocytes, storage of abnormal metabolic byproducts within myocytes or fibrotic injury are the usual causes of:

a. Dilated cardiomyopathy
b. Restrictive cardiomyopathy
c. Constrictive cardiomyopathy
d. Hypertrophic cardiomyopathy

A

b. Restrictive cardiomyopathy

How well did you know this?
1
Not at all
2
3
4
5
Perfectly
31
Q

A 27 year old female developed fever, malaise, cough and nasal congestion 2 weeks ago. She does not have chest pain nor dyspnea. Her ECG showed diffuse T wave inversions in all leads. Troponin and CK-MB are elevated. How is this patient classified in the consideration of myocarditis?

a. Possible subclinical acute myocarditis
b. Probable acute myocarditis
c. Definite myocarditis
d. Chronic myocarditis

A

b. Probable acute myocarditis

How well did you know this?
1
Not at all
2
3
4
5
Perfectly
32
Q

Peripartum cardiomyopathy develops during the last trimester of pregnancy and/or how many months after pregnancy?

a. 1
b. 2
c. 3
d. 6

A

d. 6

How well did you know this?
1
Not at all
2
3
4
5
Perfectly
33
Q

Which medication is given to patients with hypertrophic cardiomyopathy as first line agent to alleviate symptoms of LV outflow tract obstruction?

a. Disopyramide
b. ACE inhibitor
c. Diuretic
d. Beta blocker

A

d. Beta blocker

How well did you know this?
1
Not at all
2
3
4
5
Perfectly
34
Q

Which of the following is a chest x-ray finding in patients with mitral stenosis?

a. Straightening of the upper right border of the cardiac silhouette
b. Diminution of the main pulmonary arteries
c. Constriction of the upper lobe pulmonary veins
d. Posterior displacement of the esophagus by an enlarged LA

A

d. Posterior displacement of the esophagus by an enlarged LA

How well did you know this?
1
Not at all
2
3
4
5
Perfectly
35
Q

Which of the following statements correctly describes the pathologic consequences of hypertension?

a. Aggressive control of hypertension cannot reverse left ventricular hypertrophy once it sets in
b. Treatment of hypertension decreases the incidence of hemorrhagic stroke, but not of ischemic stroke
c. Risk of renal complications from hypertension is equally affected by both systolic and diastolic pressure
d. Hypertension is related to the development of dementia via beta amyloid deposition

A

a. Aggressive control of hypertension cannot reverse left ventricular hypertrophy once it sets in

How well did you know this?
1
Not at all
2
3
4
5
Perfectly
36
Q

A 45 year-old man consults for dizziness and weakness. He is a known hypertensive and is adherent to his multiple antihypertensive medications, but still records usual resting blood pressures of 150-160/100-115 mmHg. Current BP is 156/78, non-ambulatory due to bilateral lower extremity weakness. Electrolytes showed Na-140 mmol/L, K-2.9 mmol/L, Cl- 105 mmol/L, serum bicarbonate- 28 mmol/L. What is the most possible diagnosis?

a. Primary Aldosteronism
b. Renal artery stenosis
c. Pheochromocytoma
d. Renal vein thrombosis

A

a. Primary Aldosteronism

How well did you know this?
1
Not at all
2
3
4
5
Perfectly
37
Q

Which of the following is not a basic laboratory test recommended in the initial evaluation of hypertension?

a. Hba1c
b. Serum potassium
c. Serum triglyceride
d. Urinary albumin excretion

A

a. Hba1c

How well did you know this?
1
Not at all
2
3
4
5
Perfectly
38
Q

Which statement is correct regarding the side effects of ACE inhibitor or ARB therapy?

a. May cause renal insufficiency by efferent renal arteriolar vasoconstriction
b. Can cause hyperkalemia by increasing circulating aldosterone levels
c. Dry cough occurs in less than 1% of patients taking ACE inhibitors
d. Heart failure may increase the risk of renal insufficiency

A

d. Heart failure may increase the risk of renal insufficiency

How well did you know this?
1
Not at all
2
3
4
5
Perfectly
39
Q

Which of the ff causes of pulmonary hypertension is characterized by an elevated LA pressure and elevated pulmonary venous hypertension?

a. Idiopathic pulmonary arterial hypertension
b. Pulmonary hypertension associated with left heart disease
c. Pulmonary hypertension associated with lung disease
d. Pulmonary hypertension associated with chronic thromboembolic disease

A

b. Pulmonary hypertension associated with left heart disease

How well did you know this?
1
Not at all
2
3
4
5
Perfectly
40
Q

Injection drug use-associated infective endocarditis usually affects which cardiac valve?

a. mitral valve
b. tricuspid valve
c. aortic valve
d. pulmonic valve

A

b. tricuspid valve

How well did you know this?
1
Not at all
2
3
4
5
Perfectly
41
Q

Which of the following is a major criterion in the modified Duke criteria for the clinical diagnosis of infective endocarditis?

a. HACEK group organism isolated from a single blood culture
b. Fever >38.0 C
c. Increase in preexisting murmur
d. Coxiella burnetii phase I IgG antibody titer of >1:800

A

d. Coxiella burnetii phase I IgG antibody titer of >1:800

How well did you know this?
1
Not at all
2
3
4
5
Perfectly
42
Q

Mr. Cooper is a 68-year old Australian who complained of intermittent claudication was diagnosed with peripheral artery disease. One of the listed drugs he received blocks the protease activator receptor-1 that reduces acute limb ischemia but is also associated with moderate bleeding. Which drug is it?

a. Ticagleclor
b. Vorapaxar
c. Rivaroxaban
d. Pentoxyfylline

A

b. Vorapaxar

How well did you know this?
1
Not at all
2
3
4
5
Perfectly
43
Q

A 58/M, has been complaining of intermittent claudication for three months now. He is also hypertensive on Losartan 50 mg/tab but is not compliant with his medications. He is a 30-pack year smoker. Which of the following is NOT true regarding his condition?

a. This condition affects mostly the proximal vessels
b. The internal elastic lamina is preserved
c. Migratory superficial vein thrombophlebitis is seen
d. Later stages are characterized by perivascular fibrosis

A

a. This condition affects mostly the proximal vessels

How well did you know this?
1
Not at all
2
3
4
5
Perfectly
44
Q

Ella underwent a coronary angiogram for recurrent chest pains. The procedure was successful, and she stayed in the cardiac care unit for one day. When she was about to be transferred to her room, the nurse in charge noticed that her right big toe was bluish in color. When the nurse tried touching the toe, she would grimace in pain. Which of the following treatment option be given to Ella?

a. Low molecular weight heparin
b. Warfarin
c. Amputation
d. Clopidogrel

A

c. Amputation

How well did you know this?
1
Not at all
2
3
4
5
Perfectly
45
Q

Helena, a German national, has been experiencing mottled, reddish area in her legs and thighs during wintertime. It is not itchy or swollen and would go away during the summer. When she visited the Philippines, she noticed that the mottling returned when she went to Baguio for two weeks. What is the diagnosis for Helena’s condition?

a. Raynaud’s Phenomenon
b. Erythromelalgia
c. Livedo reticularis
d. Acrocyanosis

A

c. Livedo reticularis

How well did you know this?
1
Not at all
2
3
4
5
Perfectly
46
Q

Luis, 51/M, a jackhammer operator, was seen at the emergency room for pallor in his hands. The pallor would go away a few hours after rest from operating the equipment. During a routine physical in his company, his BP was 140/90, and he was given by his company physician a medication for his hypertension. However, he noticed that his hands now turned pale even if he was not operating the machinery. What antihypertensive medication could have caused this?

a. Nifedipine
b. Verapamil
c. Atenolol
d. Losartan

A

c. Atenolol

How well did you know this?
1
Not at all
2
3
4
5
Perfectly
47
Q

is a 69/M who came to emergency room for labored breathing. On history, he is a 30 pack year smoker and works as a construction worker. On PE, he has pale nailbeds, bipedal edema. Pulmonary PE showed expiratory wheezes in the bibasal lung fields. Cardiac PE showed an RV heave and S3 gallop. A 12 L ECG was requested. What ECG finding would be expected for this patient?

a. Sinus rhythm
b. Left ventricular hypertrophy
c. Right axis deviation with p pumonale
d. Paroxysmal ventricular contractions

A

c. Right axis deviation with p pumonale

How well did you know this?
1
Not at all
2
3
4
5
Perfectly
48
Q

Which of the following factors that has been shown to have worst outcomes in heart failure patients in registries?

a. Creatinine >2.75 mg/dl
b. Systolic BP >140 mmHg
c. Blood Urea Nitrogen <30 mg/dl
d. HbA1C >7.0%

A

a.Creatinine >2.75 mg/dl

How well did you know this?
1
Not at all
2
3
4
5
Perfectly
49
Q

Dr. Espiritu was called for a referral in the emergency room. The patient, a 54/M, is presented with dizziness and light-headedness. On the cardiac monitor, the tracing showed non-sustained ventricular tachycardia. Dr. Espiritu is planning to start an antiarrhythmic drug that targets the sodium ion channel with the highest potency but with slowest kinetics. Which of the following drugs describes Dr. Espiritu’s choice?

a. Flecainide
b. Quinidine
c. Procainamide
d. Lidocaine

A

Classes of antiarrhythmic actions
Class I – local anesthetic effect due to blockade of the Na current
Class Ia – moderate potency and intermediate kinetics (quinidine, procainamide)
Class Ib – low potency and rapid kinetics (lidocaine, mexiletine)
Class Ic – high potency and slowest kinetics (flecainide, propafenone)
Class II – interference with the action of catecholamines at the beta adrenergic receptor
Class III – delay of repolarization due to inhibition of K current or activation of depolarizing current
Class IV -interference with Calcium conductance

The correct answer is: Flecainide

How well did you know this?
1
Not at all
2
3
4
5
Perfectly
50
Q

Stephanie is a football athlete who was referred for cardiology clearance. She has been playing football since she was 10 years old. She has no symptoms at all. Her heart rate is slow, about 54 beats/min, but her cardiac exam is essentially normal. A 12 L ECG showed occasional no visible P waves of up to 3sec. What is the 12 L ECG reading of the patient?

a. Sinus Arrest
b. Sinus Pause
c. Type I SA block
d. Type II SA block

A

Sinus pauses and sinus arrest result from failure of the SA node to discharge, producing a pause without P waves visible on the ECG. Sinus pauses of up to 3s are common in awake athletes, and pauses of this duration or longer may be seen in asymptomatic elderly subjects
Sinus bradycardia is a rhythm driven by the SA node with a rate of < 60 beats/min. A sinus rate of < 40 beats/min in the awake state in the absence of physical condition is considered abnormal

The correct answer is: Sinus Pause

How well did you know this?
1
Not at all
2
3
4
5
Perfectly
51
Q

Karen has been experiencing light headedness for one month. She would experience “black outs” almost daily. She would have 1 episode of this black outs, but the episodes have been occurring 2-3x a day, that is why she requested for a consult. Her cardiologist advised her to have a pacemaker implanted. Which of the following is a Class I indication for pacemaker implantation?

a. Atrial fibrillation with bradycardia and pauses >5 sec
b. Mildly symptomatic patients with heart rates <40 beats/min
c. Syncope of unknown origin
d. SA node dysfunction with HR <40 beats/min on drug treatment

A

Class I indication
SA node dysfunction with symptomatic bradycardia or sinus pause
Symptomatic SA node dysfunction as a result of essential long term drug therapy with no acceptable alternatives
Symptomatic chronotropic incompetence
Atrial Fibrillation with bradycardia and pauses >5s

The correct answer is: Atrial fibrillation with bradycardia and pauses >5 sec

How well did you know this?
1
Not at all
2
3
4
5
Perfectly
52
Q

George is undergoing a laparoscopic surgery for acute cholelithiasis. He started to have right sided abdominal pain last night. His surgeon, Dr. Harris, requested for a 12 L ECG and she noticed an aberrancy in the tracing. He was immediately referred to Dr. Reyes, a cardiologist. What is the diagnosis?

a. First degree AV block
b. Mobitz Type 1 AV block
c. Mobitz Type 2 AV block
d. High Grade AV block

A

First degree AV block (PR interval > 200 ms) is a slowing of conduction through the AV junction, the site of delay is typically in the AV node but may be in the atria, bundle of His or His-Purkinje system. A wide QRS suggests delay in the distal conduction system, while a narrow QRS suggests delay in the AV node proper or in the bundle of His
The correct answer is: First degree AV block

How well did you know this?
1
Not at all
2
3
4
5
Perfectly
53
Q

Chris, 68/M, was rushed to the emergency room for persistent palpitations and light headedness. A 12 L ECG was immediately done and the ECG reader diagnosed it to be a Permanent Junctional Reciprocating Tachycardia. What is the configuration of PJRT in the 12L ECG?

a. Presence of delta waves and short PR interval
b. Narrow QRS and a right bundle branch block
c. Narrow QRS tachycardia with ventriculoatrial block
d. Negative P waves in leads II, III and aVF

A

Permanent Junctional Reciprocating Tachycardia (PJRT)
Have negative P waves in leads II, III and aVF, and slow AP conduction facilitates reentry, often leading to nearly incessant tachycardia
AV Reentry Tachycardia - The QRS is narrow or may have typical right or left bundle branch block, but without preexcitation during tachycardia
Junctional Tachycardia - Presents as a narrow QRS tachycardia, often with a ventriculoatrial block
Accessory Pathways (WPW) - ECG shows a short PR interval (<0.12 s), slurred initial portion of the QRS (delta wave), and prolonged QRS duration produced by slow conduction
The correct answer is: Negative P waves in leads II, III and aVF

How well did you know this?
1
Not at all
2
3
4
5
Perfectly
54
Q

Cooper was having palpitations and chest pains three months ago especially during exercise. However, he would be asymptomatic if he is at rest or not stressed. He underwent 24 hour Holter monitoring and results showed non-sustained ventricular tachycardia. He was given a medication to lower down the heart rate which eventually improved his condition. Two weeks ago, he started to have lethargy, cold intolerance, hair loss and noticed that he was gaining weight. What medication could have caused this?

a. Flecainide
b. Quinidine
c. Amiodarone
d. Diltiazem

A

Amiodarone and Dronaderone – amiodarone blocks multiple cardiac tonic currents and has sympatholytic activity; and is the most effective antiarrhythmic drug for suppressing ventricular arrhythmias; bradyarrhythmias are the major cardiac event, hyper or hypothyroidism are related to the iodine content of the drug. Dronaderone has structural similarities to amiodarone without the iodine moiety

The correct answer is: Amiodarone

How well did you know this?
1
Not at all
2
3
4
5
Perfectly
55
Q

Tyler, a 24/M, was rushed to emergency room for sudden loss of consciousness. He is a perfectly fit young man, no previous symptoms of chest pain, palpitations or dizziness. He was complaining that he had a slight fever and colds yesterday. One hour ago, his girlfriend saw him unconsciousness and was rushed to the ER. What would you look for in the 12 L ECG if you are suspecting Brugada Syndrome for Tyler?

a. Negative T waves in V1-V3
b. Prolonged QTc >440 ms
c. Alternating QRS morphology
d. J-point elevation

A

Brugada syndrome
Rare syndrome characterized by > 0.2 mV of ST segment elevation with a coved ST segment and negative T wave in more than one anterior precordial lead (V1-V3)
Episodes of syncope or cardiac arrest may occur during sleep or provoked by febrile illness due to polymorphic VT can occur in the absence of structural heart disease
ICD is indicated for individuals with unexplained syncope or been resuscitated from cardiac arrest
Quinidine and catheter ablation have been successful to suppress frequent episodes of VT

The correct answer is: Negative T waves in V1-V3

How well did you know this?
1
Not at all
2
3
4
5
Perfectly
56
Q

Regie was diagnosed with rheumatic heart disease since he was 12 years old. He had limited follow ups to his cardiologist because of financial constraints. One year ago, he started to have persistent tachycardia and a 12 L ECG showed Atrial Fibrillation. His cardiologist wanted to give him an antithrombin inhibitor which would lessen his INR monitoring. However, his concern was the cost of the medication. Which of the following is the prescribed drug for Regie?

a. Warfarin
b. Rivaroxaban
c. Dabigatran
d. Apixaban

A

Major options for anticoagulation are
Antithrombin inhibitor - dabigatran,
Factor Xa inhibitors rivaroxaban, apixaban and edoxaban, and the Vitamin K antagonist warfarin.
Antiplatelet agents are generally not sufficient. Warfarin is an inconvenient agent that requires several days to reach a therapeutic effect (INR >2)
The correct answer is: Dabigatran

How well did you know this?
1
Not at all
2
3
4
5
Perfectly
57
Q

Jasper is a 24 year old vegan who has been experiencing difficulty of breathing. It started 4 months ago when he started to feel tired and exhausted after work. He would be be awaken at night just to catch his breath. Last month he noticed that his ankles were swollen. Physical examination showed pale nailbeds and conjunctivae. He had a grade 2/6 systolic murmur at the 4th ICS LPSB. His cardiologist ordered a 2D echocardiogram and results showed an EF of 45%. What is the cause of the heart failure?

a. Chronic pressure overload
b. Pathologic hypertrophy
c. High output states
d. Chronic volume overload

A

Cause of heart failure: chronic anemia and possible nutritional disorders because patient is vegan.

The correct answer is: High output states

How well did you know this?
1
Not at all
2
3
4
5
Perfectly
58
Q

Quimson is a 63 year old, male, hypertensive with no maintenance anti-hypertensives. He consults for edema of 1 month duration, associated with exertional dyspnea. There is a family history of heart attack. He is also a 20 pack year smoker and occasional alcoholic drinker. He lives a sedentary lifestyle. On PE, he has bipedal edema, S3 gallop and a TR murmur. Which among the risk factors in this case is known to contribute to the development of heart failure the most?

a. Hypertension
b. Age
c. Family history
d. Smoking

A

Hypertension contributes to the development in 75% of patients, including most patients with CAD
The correct answer is: Hypertension

How well did you know this?
1
Not at all
2
3
4
5
Perfectly
59
Q

Walid, a 45 year-old male underwent an annual check up for his company. A 2D echocardiogram was done and showed LV hypertrophy with an EF of 48%. He is essentially asymptomatic. The only pertinent in the history is a family history of heart attack and his 10 pack year smoking. What compensatory mechanism would allow the functional capacity to be preserved despite of the presence of cardiac injury?

a. Decreased vasodilatory molecules such as nitric oxide
b. Activation of parasympathetic nervous system
c. Activation of the RAAS
d. Decreased myocardial contractility

A

Compensatory mechanisms that have been activated in the presence of cardiac injury which modulates LV function so that functional capacity of patient is preserved, allowing patients to remain asymptomatic or mildly symptomatic
• Activation of the RAAS and the adrenergic nervous system
• Increased myocardial contractility
• Countervailing vasodilatory molecules are activated including ANP, BNP, bradykinins, PGE2 and PGI2, and nitric oxide

The correct answer is: Activation of the RAAS

How well did you know this?
1
Not at all
2
3
4
5
Perfectly
60
Q

Hannah a 60 year old retired teacher was seen at the OPD for difficulty breathing at night. She would be awaken 3-4x at night because of having to catch her breath. Which of the following statements is consistent with the pathophysiology of her symptom?

a. Redistribution of fluid from splanchnic circulation
b. Increased sensitivity of the respiratory center to arterial PCO2
c. Reduction in stroke volume
d. Increased pressure in bronchial arteries

A

Paroxysmal Nocturnal Dyspnea (PND) refers to acute episodes of severe shortness of breath and coughing that generally occurs at night and awaken the patient from sleep. This is due to increased pressure in the bronchial arteries leading to airway compression, along with interstitial pulmonary edema that leads to increased airway resistance

The correct answer is: Increased pressure in bronchial arteries

How well did you know this?
1
Not at all
2
3
4
5
Perfectly
61
Q

Niel, 51/M, is diagnosed with Type 2 DM, and hypertension. For the latter, he is on Valsartan once a day. He had an acute MI last year and underwent PTCA for left circumflex artery. Six months ago, he experienced exertional dyspnea. The shortness of breath is progressive, he now has difficulty of breathing when walking for a few meters. He also has ankle swelling on both legs. He would also be awaken at night and has a 4 pillow orthopnea. He also feels weak most of the time. Based on the New York Heart Association, what is his heart failure classification?

a. Class I
b. Class II
c. Class III
d. Class IV

A

The NYHC:
Class I : Patients with cardiac disease but without resulting limitation of physical activity. Ordinary physical activity does not cause undue fatigue, palpitations, dyspnea or pain
Class II: Patients with cardiac disease resulting in slight limitation of physical activity. They are comfortable at rest. Ordinary physical activity results in fatigue, palpitation, dyspnea or anginal pain
Class III: Patients with cardiac disease resulting in marked limitation of physical activity. Less than ordinary activity causes fatigue, palpitation, dyspnea or anginal pain
Class IV: Patients with cardiac disease resulting in inability to carry on any physical activity without discomfort. Symptoms of heart failure or the anginal syndrome may be present even at rest. If any physical activity is undertaken, discomfort is increased

The correct answer is: Class III

How well did you know this?
1
Not at all
2
3
4
5
Perfectly
62
Q

Choi is a 69/M who came to emergency room for labored breathing. On history, he is a 30 pack year smoker and works as a construction worker. On PE, he has pale nailbeds, bipedal edema. Pulmonary PE showed expiratory wheezes in the bibasal lung fields. Cardiac PE showed an RV heave and S3 gallop. A 12 L ECG was requested. What ECG finding would be expected for this patient?

a. Sinus rhythm
b. Left ventricular hypertrophy
c. Right axis deviation, p pulmonale
d. Paroxysmal ventricular contractions

A

Cor Pulmonale 12 L ECG findings
ECG in severe pulmonary hypertension shows P pulmonale, right axis deviation and RV hypertrophy
The correct answer is: Right axis deviation, p pulmonale

How well did you know this?
1
Not at all
2
3
4
5
Perfectly
63
Q

King, a known chronic bronchial asthma patient, is seen at the OPD clinic for increased exertional dyspnea. A 2D echo was requested and showed EF of 55%. A diagnosis of HFpEF was made. Which of the following medications may be effective in improving filling pressures and RV function?

a. Candesartan
b. Isosorbide mononitrate
c. Digoxin
d. Sildenafil

A

HF with Preserved Ejection Fraction
• Control of congestion
• Stabilization of heart rate and blood pressure
• Improving exercise tolerance
• Regression of ventricular hypertrophy in hypertensive heart disease
• Use of lusitropic agents has been disappointing
• The use of Candesartan (CHARM trial) showed reduction in hospitalizations but no difference in HFpEF, same with Irbesartan
• Digitalis has no role in HFpEF
• Sildenafil improved filling pressures and RV function in a cohort of HFpEF patients with pulmonary venous hypertension

The correct answer is: Sildenafil

How well did you know this?
1
Not at all
2
3
4
5
Perfectly
64
Q

Which of the following factors that has been shown to have worst outcomes in heart failure patients in registries?

a. Creatinine >2.75 mg/dl
b. Systolic BP >140 mmHg
c. Blood Urea Nitrogen <30 mg/dl
d. HbA1C >7.0%

A

Worst outcomes in in-hospital registries:
• BUN > 43 mg/dl
• Systolic BP < 115 mmHg
• Creatinine >2.75 mg/dl
• Elevated troponin I level
The correct answer is: Creatinine >2.75 mg/dl

How well did you know this?
1
Not at all
2
3
4
5
Perfectly
65
Q

Mr. Nicolas, 69/M, is a known heart failure patient on Losartan 50 mg/tab once daily, Bisoprolol 5 mg/tab 1 tab once a day and Spironolactone 25 mg/tab 1 tab once daily. His wife noticed that during sleep, he would have episodes of no breathing followed by gasping for air, thus this consult. What is the type of respiration being described by Mr. Nicolas?

a. Biot respiration
b. Cheyne-stokes respiration
c. Central neurogenic hypoventilation
d. Kussmaul respiration

A

Cheyne-Stokes Respiration is a periodic or cyclic respiration and is present in 40% of patients with advanced HF. This is caused by an increased sensitivity of the respiratory center to arterial PCO2 and a lengthy circulatory time
Biot respiration - Biot’s respiration is an abnormal pattern of breathing characterized by groups of regular deep inspirations followed by regular or irregular periods of apnea
Kussmauls respiration - it’s characterized by a deep, rapid breathing pace throughout its duration.
Central neurogenic hypoventilation - is an abnormal pattern of breathing characterized by deep and rapid breaths at a rate of at least 25 breaths per minute.

The correct answer is: Cheyne-stokes respiration

How well did you know this?
1
Not at all
2
3
4
5
Perfectly
66
Q

Charlie, 59/M, consults for ankle swelling, bilateral for one month now. He is a known hypertensive since 2010 on Losartan 50 mg/tab once daily. Last year, he started to have intermittent chest pains especially when he is stressed, gasps for air every time he would climb the stairs of the LRT station. He was given a medication that reduces systemic vascular resistance and induces arterial vasodilation by his physician. Which of the following medications would have this particular mechanism of action?

a. Beta blocker
b. Aldosterone antagonist
c. Hydralazine
d. Nitroglycerin

A

Hydralazine reduces systemic vascular resistance and induces arterial vasodilation
Nitrates are transformed in smooth muscle cells into nitric oxide which stimulates cyclic GMP and consequent arterial-venous vasodilation
Aldosterone antagonism is associated with a reduction in mortality in all stages of symptomatic NYHA class II to IV HFrEF
• Aldosterone promote sodium imbalance
• Electrolyte imbalance
• Endothelial dysfunction
• Myocardial fibrosis
Beta blockers and RAAS blockers form the cornerstone of pharmacotherapy

The correct answer is: Hydralazine

How well did you know this?
1
Not at all
2
3
4
5
Perfectly
67
Q

Joe Santiago, a known diabetic, has been recently been diagnosed with congestive heart failure. He was experiencing worsening exertional dyspnea, 3 pillow orthopnea and bipedal edema. In the review of medications, he was on four anti-diabetic medications. Which of the following medications should avoided for heart failure patients?

a. DPP4 antagonists
b. Thiazolinediones
c. SGLT inhibitors
d. GLP-1 receptor antagonists

A

Prior studies using thiazolidinediones (activators of peroxisome proliferator activated receptors) have been associated with worsening heart failure.
Glucagon-like peptide 1 (GLP-1) agonists such as liraglutide have also been tested and do not lead to greater post-hospitalization clinical stability or worsening in heart failure.
Recently, the drug empagloflozin was tested in the EMPA-REG study and demonstrated a decrease in cardiovascular mortality
The correct answer is: Thiazolinediones

How well did you know this?
1
Not at all
2
3
4
5
Perfectly
68
Q

Rodney, 49/M obese, hypertensive and diabetic, is complaining of worsening exertional dyspnea for 8 months now. He has 3 pillow orthopnea, bidepal edema and right sided abdominal pain. He has continuous follow up with his cardiologist and the medications improved his condition. Recently, he had elevation of blood pressures of 140-150/80-85 mmHg. He sought telemedicine consult and was given a particular medicine that worsened the exertional dyspnea. Which of the following medications should be avoided in his case due to its effect destabilization of heart failure?

a. Verapamil
b. Felodipine
c. Nimodipine
d. Amlodipine

A

Amlodipine and felodipine can safely and effectively reduce blood pressure in HFrEF but do not affect morbidity, mortality or QoL
Verapamil and Diltiazem exert negative inotropic effect and destabilize previously asymptomatic patients

The correct answer is: Verapamil

How well did you know this?
1
Not at all
2
3
4
5
Perfectly
69
Q

True statement about the signs and symptoms of heart failure:

a. Orthopnea tends to appear early in the course of heart failure than exertional dyspnea
b. The coughing and wheezing of paroxysmal nocturnal dyspnea tend to persist even after the patient has assumed the upright position
c. Giant a-waves may indicate presence of tricuspid regurgitation
d. Unilateral pleural effusions can occur, and are more common on the left side because of the location of the heart on the left side of the chest

A

The correct answer is: The coughing and wheezing of paroxysmal nocturnal dyspnea tend to persist even after the patient has assumed the upright position

How well did you know this?
1
Not at all
2
3
4
5
Perfectly
70
Q

The following patient/s may be having angina pectoris or its equivalents:

a. 65 year old male, chronic smoker, who suddenly complained of acute heavy pain in the right parasternal area while doing his routine morning jog. He also complained that the pain involved the right chest up to the middle portion of his back
b. 54 year old female, uncontrolled diabetic who suddenly complained of pain in the hypogastric area occurring while she was doing grocery.
c. A 24 year old athlete who had a brief (3 minutes) episode of severe, crushing and heavy chest pain after he performed his gymnastics routine
d. All of the above patients may be experiencing angina or anginal equivalents

A

The correct answer is: A 24 year old athlete who had a brief (3 minutes) episode of severe, crushing and heavy chest pain after he performed his gymnastics routine

How well did you know this?
1
Not at all
2
3
4
5
Perfectly
71
Q

Which of the following pathophysiologic mechanism is true regarding transient ECG changes during ischemia? (P 1862)

a. Transient T wave inversion reflects non-transmural ischemia
b. Transient ST segment elevation may be seen in transmural myocardial ischemia
c. Deep ST depression reflects transmural ischemia
d. Deep T wave inversion reflects intramyocardial infarction

A

ECG changes during ischemia includes:
• Transient T wave inversion – non-transmural, intramyocardial ischemia
• Transient ST segment depression – patchy subendocardial ischemia
• ST segment elevation – reflects more severe transmural ischemia
• Repolarization abnormalities ( T wave inversion) and when severe, ST segment displacement

The correct answer is: Transient T wave inversion reflects non-transmural ischemia

72
Q

Robert was doing his brisk walking when he suddenly felt severe pain at his left chest. He suddenly placed his hand over his sternum, clutching it with a clenched fist because of discomfort. What is the medical term of Robert’s attitude of clutching the mid-chest because of pain? (p 1863)

a. Waterhammer sign
b. Osler’s sign
c. Levine’s sign
d. Turner’s sign

A

Localize pain over the sternum, sometimes with a clenched fist, to indicate a squeezing, central, substernal discomfort (Levine’s sign)
Waterhammer sign - abrupt distension and collapse of carotid arteries as a sign indicating aortic incompetence.
Osler’s sign - A patient who has a palpable, although pulseless, radial artery while the blood pressure cuff is inflated above systolic pressure
Turner’s sign - refers to bruising of the flanks, the part of the body between the last rib and the top of the hip

The correct answer is: Levine’s sign

73
Q

Dr. Ignacio was observing the treadmill exercise test of Evan, a 54/M, who complains of occasional epigastric pain whenever he goes up a flight of stairs. After 10 minutes, Dr. Ignacio stopped the exercise test. Which of the following may lead to discontinuation of the test?

a. ST elevation 0.1 mV above baseline
b. Fall of systolic blood pressure by 5 mmHg
c. Bradycardia during exercise
d. Severe shortness of breath

A

Indications for discontinuation of Treadmill Test
• chest discomfort
• severe shortness of breath
• dizziness
• severe fatigue
• ST-segment depression >0.2 mV (2 mm)
• a fall in systolic blood pressure >10 mmHg
• the development of a ventricular tachyarrhythmia
The correct answer is: Severe shortness of breath

74
Q

Geneis a 38 y.o. male, was having intermittent chest pains aggravated by running and climbing a flight of stairs and relieved by rest. He was given anti-anginal medications however, he was advised coronary arteriography. Which of the following is an indication for coronary arteriography in his case? (p 1857)

a. Diagnosed angina pectoris and are relieved by medications
b. There is a need to confirm or rule out IHD
c. Patients with low risk of sustaining coronary events
d. Patients with known angina pectoris with no known history of cardiac arrest

A

Indications Coronary arteriography is indicated in

(1) patients with chronic stable angina pectoris who are severely symptomatic despite medical therapy and are being considered for revascularization,i.e., a percutaneous coronary intervention (PCI) or coronary arterybypass grafting (CABG);
(2) patients with troublesome symptoms that present diagnostic difficulties in whom there is a need to confirm or rule out the diagnosis of IHD;
(3) patients with known or possible angina pectoris who have survived cardiac arrest;
(4) patients with angina or evidence of ischemia on noninvasive testing with clinical or laboratory evidence of ventricular dysfunction; and
(5) patients judged to be at high risk of sustaining coronary events based on signs of severe ischemia on noninvasive testing, regardless of the presence or severity of symptoms

The correct answer is: There is a need to confirm or rule out IHD

75
Q

Fr. Edgar, the parish priest, was diagnosed with angina pectoris by the primary care physician in their town. He was having occasional chest pain during his sermons but would be relieved whenever he sits down. He was given an unknown medication which relieved his symptoms. However, he started to have dizziness especially when standing up, headaches, and palpitations whenever he stops his medication. Which of the following may have caused the symptoms? (p 1862)

a. Metoprolol
b. Diltiazem
c. Amlodipine
d. Nitroglycerin

A

The correct answer is: Nitroglycerin

76
Q

Dr. Reyes was working on an alternative medication for chest pains. The medication’s mechanism of action opens ATP-sensitive potassium channels in myocytes, which leads to a reduction of free intracellular ions. Which of the following medications has the abovementioned mechanism of action? (p 1862)

a. Nicorandil
b. Ivabradine
c. Ranolazine
d. Sacubutril

A

Another class of agents opens ATP-sensitive potassium channels in myocytes, leading to a reduction of free intracellular calcium ions. The major drug in this class is nicorandil.
Ranolazine, a piperazine derivative, may be useful for patients with chronic angina despite standard medical therapy. Its antianginal action is believed to occur via inhibition of the late inward sodium current (INa)
Ivabradine (2.5–7.5 mg orally twice daily) is a specific sinus node inhibiting agent that may be helpful for preventing cardiovascular events in patients with IHD who have a resting heart rate ≥70 beats/min (alone or in combination with a beta blocker) and LV systolic dysfunction.
The correct answer is: Nicorandil

77
Q

What among the statements is an indication for percutaneous coronary intervention (PCI)?

a. Left main coronary artery disease
b. Left ventricular Ejection fraction <50%
c. Two-vessel coronary artery disease
d. Presence of diabetes mellitus

A

It is now recommended that patients with an unacceptable level of angina despite optimal medical management be considered for coronary revascularization. Patients with single- or two-vessel disease with normal LV function and anatomically suitable lesions ordinarily are advised to undergo PCI.
The correct answer is: Two-vessel coronary artery disease

78
Q

Which of the following is true regarding cardiac biomarkers?

a. At least one value is above the 90th percentile of the upper reference limit
b. Creatine phosphokinase rises within 2-4 hours of the event
c. The MM isoenzyme of the creatine phosphokinase is considerably specific
d. A relative index of CK-MB to CK activity >=2.5 suggestive of myocardium

A

CK rises within 4–8 h and generally returns to normal by 48–72 h (Fig. 269-3). An important drawback of total CK measurement is its lack of specificity for STEMI, as CK may be elevated with skeletal muscle disease or trauma, including intramuscular injection. The MB isoenzyme of CK has the advantage over total CK that it is not present in significant concentrations in extracardiac tissue and, therefore, is considerably more specific. However, cardiac surgery, myocarditis, and electrical cardioversion often result in elevated serum levels of the MB isoenzyme. A ratio (relative index) of CK-MB mass to CK activity ≥2.5 suggests but is not diagnostic of a myocardial rather than a skeletal muscle source for the CK-MB

The correct answer is: A relative index of CK-MB to CK activity >=2.5 suggestive of myocardium

79
Q

What is the definition of a PCI-related myocardial infarction?

a. Elevation of cardiac troponin >2x the upper limit of normal baseline values
b. Elevation of cardiac troponin >90th percentile above the upper limit of normal baseline values
c. Cardiac Troponin >20% above if the baseline values are elevated
d. Cardiac Troponin >95th percentile above if the baseline values are elevated

A

Myocardial infarction associated with PCI is arbitrarily defined by elevation of cardiac troponin (cTn) values >5 Å~ 99th percentile upper reference limit (URL) in patients with normal baseline values (≤99th percentile URL) or a rise of cTn values >20% if the baseline values are elevated and are stable or falling.
In addition, either (i) symptoms suggestive of myocardial ischemia, or (ii) new ischemic ECG changes or new LBBB, or (iii) angiographic loss of patency of a major coronary artery or a side branch or persistent slow or no flow or embolization, or (iv) imaging demonstration of new loss of viable myocardium or new regional wall motion abnormality is required.

The correct answer is: Cardiac Troponin >20% above if the baseline values are elevated

80
Q

Andy was rushed to the emergency room for chest pains. Unfortunately, the hospital he was brought initially, does not have a cardiac care facility. How fast must Andy be transferred to a cardiac care facility with a goal of initiating PCI?

a. 60 min
b. 90 min
c. 120 min
d. 150 min

A

This may involve transfer from a non-PCI hospital to one that is PCI capable, with a goal of initiating PCI within 120 min of first medical contact

The correct answer is: 120 min

81
Q

A 32/F, non-diabetic, non-hypertensive, diagnosed case of Cervical Cancer Stage III presented at the ER for 1 week history of exertional dyspnea followed by a 2 hour history of sudden, crushing chest pain. Her ECG showed acute anterior wall myocardial infarction. Further history showed one month of incessant vaginal bleeding. Her hemoglobin is only 5 g/L. Using the Universal Definition of Myocardial Infarction, what type of myocardial infarction does this patient most likely have? (HPIM 20th Ed. P. 1877)

a. Type 1
b. Type 2
c. Type 3
d. Type 4

A

Type 1 – Spontaneous Myocardial Infarction
Type 2 – MI sec to an ischemic imbalance
Type 3 – MI resulting in Death when values are unavailable
Type 4 - MI related to PCI/Stent

The correct answer is: Type 2

82
Q

The anatomical site that determines most of the peripheral vascular resistance:

a. Medium-sized arteries and arterioles
b. Small and medium-sized arteries
c. Small arteries and arterioles
d. Arterioles and venoules

A

The correct answer is: Small arteries and arterioles

83
Q

Which of the following statements correctly describes the pathologic consequences of hypertension?

a. Aggressive control of hypertension cannot reverse left ventricular hypertrophy once it sets in
b. Treatment of hypertension decreases the incidence of hemorrhagic stroke, but not of ischemic stroke
c. Risk of renal complications from hypertension is equally affected by both systolic and diastolic pressure
d. Hypertension is related to the development of dementia via beta amyloid deposition

A

The correct answer is: Hypertension is related to the development of dementia via beta amyloid deposition

84
Q

Which of the following is true regarding genetic considerations of hypertension?

a. Genetic determinants account for 25% of all blood pressure variants.
b. The gene coding for alpha adducin is associated with decreased sodium reabsorption
c. Genetic determinants have been linked to CHD and stroke
d. Detailed genome-wide epigenetic modifications in hypertension have pinpointed specific genes for hypertension

A

Specific genetic variants have been linked to CHD and stroke.
Identified genetic determinants account for ~1% of blood pressure variance
Genes that encode components of the renin-angiotensin-aldosterone system, atrial natriuretic peptide, the beta-2 adrenoreceptor, and alpha adducin (associated with increased renal tubular reabsorption of sodium).
Current results of detailed genome-wide epigenetic modifications of DNA are limited and conflicting.
The correct answer is: Genetic determinants have been linked to CHD and stroke

85
Q

Chung, a Chinese national, visited the out-patient clinic for hypertension. She is 63 years old and a retired teacher. Her blood pressure readings were 150-160/90 mmHg and is associated with nape pains and palpitations. When asked about her diet, she loves to eat a typical Chinese meal heavy on salt. Which of the following is true regarding salt-dependent hypertension? (p 1892)

a. This is caused by an increased capacity of the kidney to excrete salt
b. There is increased production of mineralocorticoid resulting to sodium secretion
c. There is decreased neural activity of the kidney
d. There is increased activity of the renin-angiotensin system

A

NaCl-dependent hypertension may be a consequence of a decreased capacity of the kidney to excrete sodium, due either to intrinsic renal disease or to increased production of a salt-retaining hormone (mineralocorticoid) resulting in increased renal tubular reabsorption of sodium.
Renal tubular sodium reabsorption also may be augmented by increased neural activity to the kidney. In each of these situations,a higher arterial pressure may be required to achieve sodium balance.
Conversely, salt-wasting disorders are associated with low blood pressure levels. ESRD is an extreme example of volume-dependent hypertension.
In ~80% of these patients, vascular volume and hypertension can be controlled with adequate dialysis; in the other 20%, the mechanism of hypertension is related to increased activity of the renin-angiotensin system and is likely to be responsive to pharmacologic blockade of renin-angiotensin.
The correct answer is: There is increased activity of the renin-angiotensin system

86
Q

George is a 23 year-old basketball player who complains of nape pains and elevated BP. Work up has been done and secondary causes have been ruled out. He is very active and watches his diet very well. He decided to join this experiment which measures his sympathetic nervous system, and the results showed that his BP elevation is caused by hyperactivity of the SNS. Which of the following is FALSE regarding the impact of the autonomic nervous system? (p 1892)

a. Upregulation of receptors may be a consequence of high levels of catecholamines
b. Increased firing in stretch-sensitive sensory nerve endings in the carotid sinuses may elevated blood pressure
c. Norepinephrine, epinephrine and dopamine all play important roles in SNS regulation
d. Both normal weight and obese individuals may be associated with increased sympathetic outflow.

A

Downregulation of receptors maybe a consequence of sustained high levels of catecholamines and providesan explanation for decreasing responsiveness, or tachyphylaxis, to catecholamines.
Rest are true
The correct answer is: Upregulation of receptors may be a consequence of high levels of catecholamines

87
Q

Which of the following is not a stimuli for renin secretion?

a. Decreased NaCL transport in the distal portion of the thick ascending limb of the loop of Henle
b. Angiotensin II inhibition of renin secretion on the JG cells
c. Decreased pressure or stretch in the renal afferent arteriole
d. SNS stimulation of renin-secreting cells via beta1 receptors

A

Three primary stimuli for Renin production:
- decreased NaCl transport in the distal portion of the thick ascending limb of the loop of Henle that abuts the corresponding afferent arteriole (macula densa),
- decreased pressure or stretch within the renal afferent arteriole (baroreceptor mechanism), and
- sympathetic nervous system stimulation of renin-secreting cells via β1 adrenoreceptors.
The correct answer is: Angiotensin II inhibition of renin secretion on the JG cells

88
Q

In the Hypertension in the Very Elderly clinical trial, the results using a diuretic was very promising in lowering blood pressure. Dr. Ramos decided to give indapamide to his 84 year old patient. However, four days prior to taking the medication, his patient was rushed to the eergency room for dehydration. Dr. Ramos was looking for an alternative medication for the BP elevation. Which of the following pathophysiologic consequences of hypertension would Dr. Ramos consider for his patient?

a. Increased Renin-angiotensin-aldosterone activation
b. Hyperactive autonomic nervous system
c. Arterial stiffness
d. Salt dependent hypertension

A

An association between arterial stiffness and hypertension is well established. A stiffened vasculature is less able to buffer shorttermalterations in flow. Although it has been assumed that arterial stiffness is a manifestation of hypertension, recent evidence suggests that vascular stiffness may also represent a cause of hypertension.

The correct answer is: Arterial stiffness

89
Q

Tony, a 55 year old male noticed that his BP readings were sustained at 140/90 mmHg since last year. He is not complaining of any symptoms. Recently, he stopped exercising because of work and gained a few extra pounds. Laboratory results showed proteinuria. Which of the following kidney structures is primarily affected in his case to produce proteinuria?

a. Preglomerular arterioles
b. Postglomerular arterioles
c. Intraglomerular cells
d. Glomerular capillary

A

Atherosclerotic, hypertension-related vascular lesions in the kidney primarily affect preglomerular arterioles, resulting in ischemic changes in the glomeruli and postglomerular structures. Glomerular injury also may be a consequence of direct damage to the glomerular capillaries due to glomerular hyperperfusion. Studies of hypertension-related renal damage, primarily in experimental animals, suggest that loss of autoregulation of renal blood flow at the afferent arteriole results in transmission of elevated pressures to an unprotected glomerulus with ensuing hyperfiltration, hypertrophy, and eventual focal segmental glomerular sclerosis.
The correct answer is: Preglomerular arterioles

90
Q

Myra, a 30 year old school teacher was rushed to the emergency room for high grade fever. She has experiencing occasional tremors, diarrhea, weight loss and hair loss. Which blood pressure characteristic is to be expected in this case?

a. Elevated systolic and diastolic BP
b. Elevated diastolic BP
c. Widened pulse pressure
d. High mean arterial pressure

A

Systolic Hypertension with Wide Pulse Pressure

  1. Decreased vascular compliance (arteriosclerosis)
  2. Increased cardiac output
    a. Aortic regurgitation
    b. Thyrotoxicosis
    c. Hyperkinetic heart syndrome
    d. Fever
    e. Arteriovenous fistula
    f. Patent ductus arteriosus

The correct answer is: Widened pulse pressure

91
Q

Dorie, an 84 year old, was rushed to the emergency room for sudden elevation of blood pressure. She has been hypertensive since she was 80 years old and was controlled on amlodipine 5 mg once daily. However, her BP reading last year started to increase some more. Two weeks ago, she went to her cardiologist. She was given an unrecalled medication. However, three days on the medication, her BP shot up to 160 systolic with associated nape pains. What condition may have caused this? (p 1897)

a. Chronic kidney disease
b. Pheochromocytoma
c. Isolated systolic hypertension
d. Renovascular hypertension

A

Renovascular hypertension should be considered in patients with other evidence of atherosclerotic vascular disease. Severe or refractor hypertension, recent loss of hypertension control or recent onset of moderately severe hypertension, and unexplained deterioration of renal function or deterioration of renal function associated with an ACE inhibitor should raise the possibility of renovascular hypertension. Approximately 50% of patients with renovascular hypertension have an abdominal or flank bruit, and the bruit is more likely to be hemodynamically significant if it lateralizes or extends throughout systole into diastole.
The correct answer is: Renovascular hypertension

92
Q

Jane has been experiencing bilateral leg weakness. Yesterday she had another attack of leg weakness and was immediately brought to the company clinic. Her BP reading was 148/86 mmHg at the clinic. She was given Gatorade which improved her weakness and was advised to see a specialist. Her laboratory results showed a low potassium of 3.0 mg/dl. The specialist decided to measure the plasma aldosterone and renin activity. Which of the following results is highly sensitive to her diagnosis?

a. Plasma aldosterone concentration/plasma renin activity (PAC/PRA) of >10:1
b. PAC/PRA >20:1
c. PAC/PRA >30:1
d. PAC/PRA >40:1

A

The ratio of plasma aldosterone to PRA (PA/PRA) is a useful screening test. These measurements preferably are obtained in ambulatory patients in the morning. A ratio >30:1 in conjunction with a plasma aldosterone concentration >555 pmol/L (>20 ng/dL) reportedly has a sensitivity of 90% and a specificity of 91% for an aldosterone-producing adenoma.
The correct answer is: PAC/PRA >30:1

93
Q

Ronald has been having headaches, flushing and elevated blood pressures. He would have several visits to the emergency room for BP as high as 200 mmHg systolic. He is already on four antihypertensive medications, but his BP is still elevated. Which of the following medication is the most appropriate for Ronald?

a. Phentolamine
b. Clonidine
c. Alpha methyldopa
d. Guanfacine

A

Catecholamine-secreting tumors are located in the adrenal medulla (pheochromocytoma) or in extra-adrenal paraganglion tissue (paraganglioma) and account for hypertension in ~0.05% of patients. If unrecognized, pheochromocytoma may result in lethal cardiovascular consequences. Medical treatment involves alpha blockers, and surgical management is the definitive treatment.

The correct answer is: Phentolamine

94
Q

Which of the following genetic causes is associated with polycystic kidney disease?

a. Mutation in chromosome 16
b. Mutation in the RET protooncogene
c. Random mutation in chromosome 10
d. Mutation in chromosome 8q21

A

Polycystic kidney disease - Mutations in the PKD1 gene on chromosome 16 and PKD2 gene on chromosome 4
Pheochromocytoma - Mutations in the RET protooncogene
17α-hydroxylase deficiency - Random mutations of the CYP17 gene on chromosome 10
11β-hydroxylase deficiency - Mutations of the CYP11B1 gene on chromosome 8q21-q22

The correct answer is: Mutation in chromosome 16

95
Q

Marlon is a newly diagnosed hypertensive. His BP readings range from 140-150/70-80 mmHg. He had a high predisposition to hypertension as both his parents were hypertensive. During his consult, he asked what lifestyle change he needs to implement to help in lowering his BP. Which of the following is correct?

a. Attain a BMI of <30 kg/m2
b. Consume <2 drinks/day in women
c. Salt reduction of <8 g NaCl/day
d. Brisk walking 30 min/day

A

Weight reduction – attain and maintain < 25 kg/m2
Dietary salt reduction - < 6 g NaCl
Diet rich in potassium, vegetables, low fat dairy (DASH diet)
Regular aerobic exercise for 30 min/day
For those who drink alcohol, consume ≤2 drinks/d in men and ≤1 drink/d in women

The correct answer is: Brisk walking 30 min/day

96
Q

Dave is a 49 y.o. male with a difficult-to-treat hypertension. He is presently on four anti-hypertensive medications and with BP readings 130-140/70-75 mmHg. However, two weeks ago, his right big toe became painful and inflamed. Which of the following medications may have caused this?

a. Spironolactone
b. Amiloride
c. Chlorthalidone
d. Triamterene

A

Thiazide and loop diuretics may lead to gout.
More common side effects of aldosterone blockers and K- sparing diuretics are renal failure and hyperkalemia
The correct answer is: Chlorthalidone

97
Q

Leo is a 54 year old man who complains of intermittent calf pains whenever he exercises in the morning for the past 6 months. He describes it as cramping and aching sensation when he exercises and would be relieved by rest. The pain was increasing in intensity, thus the consult. What physical finding is commonly seen in this condition?

a. Increased pulses in the popliteal area
b. Gain in muscle mass
c. Hair loss
d. Absence of bruits distal to obstruction

A

Important physical findings of PAD include decreased or absent pulses distal to the obstruction, the presence of bruits over the narrowed artery, and muscle atrophy. With more severe disease, hair loss, thickened nails, smooth and shiny skin, reduced skin temperature, and pallor or cyanosis are common physical signs

The correct answer is: Hair loss

98
Q

Ming, 64/F, was seen in the out patient clinic for left leg pain. She had been complaining of the leg pain whenever she starts walking and would be relieved by rest. However, the pain was increasingly bothersome so she went in for a consult. The primary care physician measured both brachial and ankle blood pressures. What value of the ankle:brachial index (ABI) would be indicative of valvular calcification?

a. <0.90
b. 0.91- 0.99
c. 1.0 – 1.40
d. >1.40

A

The ratio of the ankle and brachial artery pressures (termed the ankle:brachial index, or ABI) is 1.00–1.40 in normal individuals. ABI values of 0.91–0.99 are considered “borderline,” and those <0.90 are abnormal and diagnostic of PAD. ABIs >1.40 indicate noncompressible arteries secondary to vascular calcification

The correct answer is: >1.40

99
Q

Mr. Cooper is a 68 year old Australian was diagnosed with peripheral artery disease for intermittent claudication in both his legs. He was given a medication that blocks the protease activator receptor-1 that reduces acute limb ischemia but is also associated with moderate bleeding. Which of the following is the drug given to Mr. Cooper? (p. 1924)

a. Ticagleclor
b. Vorapaxar
c. Rivaroxaban
d. Pentoxyfylline

A

The ratio of the ankle and brachial artery pressures (termed the ankle:brachial index, or ABI) is 1.00–1.40 in normal individuals. ABI values of 0.91–0.99 are considered “borderline,” and those <0.90 are abnormal and diagnostic of PAD. ABIs >1.40 indicate noncompressible arteries secondary to vascular calcification

The correct answer is: Vorapaxar

100
Q

Janise 24/F complains of elevated blood pressures of 140 mmHg systolic. Her BP started to be elevated two years ago. She had no other symptoms except of intermittent calf pains when walking. The interventional cardiologist decided to do an angiogram on her iliac arteries and identified a “string of beads” appearance. Which of the following is true regarding her condition? (p 1925)

a. It is characterized by alternating areas of thinned media and fibromuscular ridges
b. Polymorphonuclear leukocytes infiltrate the walls of small and medium-sized arteries and veins
c. Arterial occlusion results in the sudden cessation of blood flow to an extremity
d. It involves compression of the neurovascular bundle at the legs which leads to the claudication

A

Fibromuscular dysplasia is a hyperplastic disorder that affects medium-size and small arteries. It occurs predominantly in females and usually involves the renal and carotid arteries but can affect extremity vessels such as the iliac and subclavian arteries. The histologic classification includes intimal fibroplasia (also classified as focal), medial dysplasia (multifocal), and adventitial hyperplasia. Medial dysplasia is subdivided into medial fibroplasia, perimedial fibroplasia, and medial hyperplasia. Medial fibroplasia is the most common type and is characterized by alternating areas of thinned media and fibromuscular ridges.
The correct answer is: It is characterized by alternating areas of thinned media and fibromuscular ridges

101
Q

Jun, 58/M, has been complaining of intermittent calf pains since three months ago. He noticed that whenever he climbs a flight of stairs riding the MRT, he would experience the pain but would be relieved by rest. He is also hypertensive on Losartan 50 mg/tab but is not compliant with his medications. He is also a 30 pack year smoker. Which of the following is NOT true regarding his condition?

a. This condition affects mostly the proximal vessels
b. The internal elastic lamina is preserved
c. Migratory superficial vein thrombophlebitis is seen
d. Later stages are characterized by perivascular fibrosis

A
Thromboangiitis obliterans (Buerger’s disease) is an inflammatory occlusive vascular disorder involving small and medium-size arteries and veins in the distal upper and lower extremities. Cerebral, visceral, and coronary vessels may be affected rarely. This disorder develops most frequently in men <40 years of age. The prevalence is higher in Asians and individuals of Eastern European descent. Although the cause of thromboangiitis obliterans is not known, there is a definite relationship to cigarette smoking in patients with this disorder.
The internal elastic lamina is preserved, and a cellular, inflammatory thrombus develops in the vascular lumen. As the disease progresses, mononuclear cells, fibroblasts, and giant cells replace the neutrophils. Later stages are characterized by perivascular fibrosis, organized thrombus, and recanalization.

The correct answer is: This condition affects mostly the proximal vessels

102
Q

Mrs. Santos was rushed immediately to the emergency room for severe pain in the left foot. She is 68 years old, suffered a heart attack last year but was lost to follow up because of financial constraints. A few hours ago, she was doing household chores when she suddenly felt pain in her left foot. When the resident examined her, the left foot was painful, numb and cold to touch. What treatment should be given to Mrs. Santos? (p 1926)

a. Abstinence from tobacco is the treatment of choice
b. Thrombolytic therapy is most effective even if occlusion is chronic
c. Limb amputation is performed when the limb is nonviable
d. Surgical revascularization is preferred within 48 hours to prevent limb loss

A

Once the diagnosis of acute limb ischemia is made, the patient should be anticoagulated with intravenous heparin to prevent propagation of the clot and recurrent embolism. Catheter-directed thrombolysis/ thrombectomy, surgical thromboembolectomy, and arterial bypass procedures are used to restore blood flow to the ischemic extremity promptly, particularly when a large proximal vessel is occluded.
Intraarterial thrombolytic therapy with recombinant tissue plasminogen activator, reteplase, or tenecteplase is most effective when acute arterial occlusion is recent (<2 weeks) and caused by a thrombus in an atherosclerotic vessel, arterial bypass graft, or occluded stent.
Surgical revascularization is preferred when restoration of blood flow must occur within 24 h to prevent limb loss or when symptoms of occlusion have been present for >2 weeks.
Amputation is performed when the limb is not viable, as characterized by loss of sensation, paralysis, and the absence of Doppler-detected blood flow in both arteries and veins.

The correct answer is: Limb amputation is performed when the limb is nonviable

103
Q

Ella underwent a coronary angiogram for recurrent chest pains. The procedure was successful, and she was stayed in the cardiac care unit for one day. When she was about to be transferred to her room, the nurse in charge noticed that her right big toe was bluish in color. When the nurse tried touching the toe, she would grimace in pain. Which of the following treatment option be given to Ella? (p 1927)

a. Low molecular weight heparin
b. Warfarin
c. Amputation
d. Clopidogrel

A

Once the diagnosis of acute limb ischemia is made, the patient should be anticoagulated with intravenous heparin to prevent propagation of the clot and recurrent embolism. Catheter-directed thrombolysis/ thrombectomy, surgical thromboembolectomy, and arterial bypass procedures are used to restore blood flow to the ischemic extremity promptly, particularly when a large proximal vessel is occluded.
Intraarterial thrombolytic therapy with recombinant tissue plasminogen activator, reteplase, or tenecteplase is most effective when acute arterial occlusion is recent (<2 weeks) and caused by a thrombus in an atherosclerotic vessel, arterial bypass graft, or occluded stent.
Surgical revascularization is preferred when restoration of blood flow must occur within 24 h to prevent limb loss or when symptoms of occlusion have been present for >2 weeks.
Amputation is performed when the limb is not viable, as characterized by loss of sensation, paralysis, and the absence of Doppler-detected blood flow in both arteries and veins.

The correct answer is: Amputation

104
Q

Romy is a 24/M star runner in the university. During practices, he would occasionally experience bilateral calf pains, but the pain would go away when he rests. The pain was increasingly unbearable, so he decided to have it checked. The school physician noted essentially lower extremity findings. However, the doctor was able to elicit the pain when he performed ankle dorsiflexion to Romy. What could be the diagnosis?

a. Popliteal artery aneurysm
b. Outlet compression syndrome
c. Arteriovenous fistula
d. Popliteal artery entrapment

A

Popliteal artery entrapment typically affects young athletic men and women when the gastrocnemius or popliteus muscle compresses the popliteal artery and causes intermittent claudication. Thrombosis, embolism, or popliteal artery aneurysm may occur
Patients with popliteal artery aneurysms often have aneurysms of other arteries, especially the aorta. The most common clinical presentation is limb ischemia secondary to thrombosis or embolism
Abnormal communications between an artery and a vein, bypassing the capillary bed, may be congenital or acquired
Outlet compression syndrome is a symptom complex resulting from compression of the neurovascular bundle (artery, vein, or nerves)

The correct answer is: Popliteal artery entrapment

105
Q

Nick was born with a pulsatile birthmark in his right thigh. He did not notice it until one month ago when the birthmark was beginning to increase in size. When he touches it, he would feel a thrilling sensation. His right leg was beginning to swell and the right leg was beginning to look pale. He sought consult to his primary care physician who referred him to a vascular surgeon. What is the best option for treatment for Nick? (p 1928)

a. Conservative treatment
b. Surgical treatment of the fistula
c. Radiotherapy
d. Chemoembolization of the AV fistula

A

The correct answer is: Conservative treatment

106
Q

Charo, 36/F, would notice that her hands would become pale whenever she is stressed or when she goes to a cold climate country. It would be whitish and feels cold. But when she relaxes or wears gloves, her hands would finally turn reddish at first and eventually would become “normal”. She is afraid that one day her fingers will just fall off. She was given a medication that made the pallor less tolerable. Which of the following medications has been shown to be effective to decrease the frequency and severity of her disease? (p 1928)

a. Losartan
b. Nifedipine
c. Verapamil
d. Atenolol

A

Drug treatment should be reserved for severe cases of Raynaud’s phenomenon. Dihydropyridine calcium channel antagonists such as nifedipine, isradipine, felodipine, and amlodipine decrease the frequency and severity of Raynaud’s phenomenon. Diltiazem may be considered but is less effective. The postsynaptic α1-adrenergic antagonist prazosin has been used with favorable responses; doxazosin and terazosin may also be effective.
The correct answer is: Nifedipine

107
Q

Helena, a German national, has been experiencing mottled, reddish area in her legs and thighs during wintertime. It is not itchy or swollen and would go away during the summer. When she visited the Philippines, she noticed that the reddish patch returned when she went to Baguio for two weeks. What is the diagnosis for Helena’s condition?

a. Raynaud’s Phenomenon
b. Erythromelalgia
c. Livedo reticularis
d. Acrocyanosis

A

Livedo Reticularis - localized areas of the extremities develop a mottled or rete (netlike) appearance of reddish to blue discoloration The mottled appearance may be more prominent after cold exposure
Acrocyanosis there is arterial vasoconstriction and secondary dilation of the capillaries and venules with resulting persistent cyanosis of the hands and, less frequently, the feet.
Erythomelalgia is characterized by burning pain and erythema of the extremities. The feet are involved more frequently than the hands, and males are affected more frequently than females.
Raynaud’s phenomenon is characterized by episodic digital ischemia, manifested clinically by the sequential development of digital blanching, cyanosis, and rubor of the fingers or toes after cold exposure and subsequent rewarming.
The correct answer is: Livedo reticularis

108
Q

Luis, 51/M, a construction worker, was seen at the emergency room for pallor in his hands. He would notice that his hands would become pale whenever he would operate the jackhammer in the construction site. The pallor would go away a few hours after resting. During a routine physical in his company, his BP was 140/90, and he was given by his company physician a medication for his hypertension. However, he noticed that his hands turned white even if he was not operating the machinery. What antihypertensive medication could have caused this? (p 1929)

a. Nifedipine
b. Verapamil
c. Atenolol
d. Losartan

A

Several drugs have been causally implicated in Raynaud’s phenomenon. They include ergot preparations, methysergide, β-adrenergic receptor antagonists, and the chemotherapeutic agents bleomycin, vinblastine, cisplatin, and gemcitabine.

The correct answer is: Atenolol

109
Q

Cora is the medical intern who was assigned to take care of the IM patients. She asked her attending physician regarding pain, tingling sensation of the right arm and pallor pf her right hand when she would reach for objects or do arm exercises. Which of the following diagnostic results would be appropriate to clinch her diagnosis?

a. Chest x-ray will indicate the absence of cervical ribs
b. Electromyogram have high sensitivity in neurogenic compression
c. Duplex ultrasonography may demonstrate arterial compression
d. Contrast angiography has no role in the diagnosis

A

In thoracic outlet compression syndrome, a chest x-ray will indicate the presence of cervical ribs. Duplex ultrasonography, MRA, and contrast angiography can be performed during provocative maneuvers to demonstrate thoracic outlet compression of the subclavian artery.
Neurophysiologic tests such as the electromyogram, nerve conduction studies, and somatosensory evoked potentials may be abnormal if the brachial plexus is involved, but the diagnosis of neurogenic thoracic outlet syndrome is not necessarily excluded if these tests are normal owing to their low sensitivity.
The correct answer is: Duplex ultrasonography may demonstrate arterial compression

110
Q

Based on the latest Philippine CPG on dyslipidemia, what is the optimal level of LDL-C?

a. <50 mg/dl
b. <100 mg/dl
c. <150 mg/dl
d. <120 mg/dl

A

The correct answer is: <100 mg/dl

111
Q

Latest studies have shown that apolipoproteins may have a role in cholesterol formation. In the cholesterol synthesis pathway, which of the following apolipoproteins inhibit lipoprotein lipase?

a. apoB-100
b. apoC-II
c. apoE
d. apoC-III

A

Apolipoproteins (apo) C-I and C-III are known to inhibit lipoprotein lipase (LPL) activity, but the molecular mechanisms for this remain obscure.
The correct answer is: apoC-III

112
Q

Which statin causes the least amount of LDL reduction?

a. Rosuvastatin 20 mg
b. Atorvastatin 40 mg
c. Simvastatin 20mg
d. Fluvastatin 40 mg

A

The correct answer is: Fluvastatin 40 mg

113
Q

James had another heart attack two weeks ago. He underwent PTCA and was released on different medications. His LDL-c was already at 70 mg/dl, however, his HDL-c was at 30 mg/dl. His cardiologist decided to put him on Niacin. What is the most common side effect of niacin that we need to warn our patients?

a. Myalgia
b. Flushing
c. Increase cholesterol level
d. Joint pains

A
Niacin have been shown to cause the following adverse events:
•	Severe skin flushing combined with dizziness.
•	Rapid heartbeat.
•	Itching.
•	Nausea and vomiting.
•	Abdominal pain.
•	Diarrhea.
•	Gout.
•	Liver damage
The correct answer is: Flushing
114
Q

Which of the following is part of the criteria for metabolic syndrome under the Harmonizing Definition Criteria for Metabolic Syndrome?

a. Fasting triglyceride level of >180 mg/dL or medication
b. HDL cholesterol level <50 mg/dL for men
c. Fasting plasma glucose level of >100 mg/dL
d. Blood pressure of >130 mmHg systolic

A

The correct answer is: Blood pressure of >130 mmHg systolic

115
Q

Coco is a 64/F who suffered a heart attack three months ago. She underwent a successful PTCA and is ready to be discharged. Her cardiologist decided to put her on ezetimibe on top of a high dose atorvastatin to prevent future cardiovascular event. Based on the IMPROVE-IT trial, when is it expected to see the use of ezetimibe on top of a statin therapy in preventing cardiovascular events?

a. After 1 year of use
b. After 2 years of use
c. After 3 years of use
d. After 4 years of use

A

In IMPROVE-IT, the between-group difference in LDL cholesterol was calculated as the difference in the observed LDL cholesterol level in patients from whom blood samples were obtained at 1 year.
The correct answer is: After 1 year of use

116
Q

Jenny has bilateral masses on her elbows. The masses were soft, doughy and movable. She had them checked by her primary care physician. On history, her sister also has the same masses but is located on her knees. Their father died of a heart attack when he was only 48 years old. Based on the Dutch Lipid Score, what level would indicate that Jenny has DEFINITE familial hypercholesterolemia?

a. Score of 3
b. Score of 5
c. Score of 7
d. Score of 9

A

Refer to the Dutch Lipid Network criteria on the diagnosis of heterozygous familial hypercholesterolemia
The correct answer is: Score of 9

117
Q

Mary suffered a heart attack four months ago. She went home a few days after a successful bypass. Dr. Gonzales, her cardiologist, placed on atorvastatin 80 mg/dl. A repeat LDL-c on follow up showed a level of 140mg/dl. Which of the following drugs may be added to lower her LDL-c to target of < 100 mg/dl?

a. Niacin
b. PCSK-9 inhibitors
c. CETP inhibitors
d. Clofibrate

A

Proprotein Convertase Subtilisin/Kexin Type 9 inhibitors (PCSK9-I), are available worldwide as an adjunct to statins in lowering cholesterol levels. These are monoclonal antibodies which bind to and inactivate an enzyme in the liver called proprotein convertase subtilisin kexin 9 (PCSK-9), which inactivates the need receptors on the liver cell surface that transport LDL-C into the liver for metabolism. Evolucumab is already available in the country, while Alirocumab, the other PCSK-9 inhibitor, is only available in other countries. Bococizumab showed no benefit in cardiovascular morbidity and mortality in phase 3 clinical trials.
The use of PCSK-9 inhibitors can lower LDL-C by < 70mg/dl (59% lower than the baseline) and can go as low as 30 mg/dl.

The correct answer is: PCSK-9 inhibitors

118
Q

Mario, a 48 year old engineer has the following annual examination findings: LDL-c, 138 mg/dl. He is sedentary lifestyle as he is confined in the office and has recently gained two pounds but is not obese (BMI 24 kg/m2). His father died of a heart attack at 50 years old. His diet consists of mostly salty food. Which of the following risk factors that is present in Mario may prompt his physician to start statin therapy?

a. BMI of 24 kg/m2
b. Sedentary lifestyle
c. Family history
d. Diet

A

In the 2015 CPG, we identified Risk Factor Counting as the method in identifying the risk of the Filipino individual for cardiovascular disease. We continue to recommend this as the available risk factor scores advocated by the different guidelines (eg: ASCVD score) are not validated for the Filipino individuals. The following risk factors were identified and if the individual has more than two (2) risk factors, the individual has a risk for cardiovascular disease:
male sex, postmenopausal women, smoker, hypertension, BMI > 25 kg/m2, family history of premature CHD, proteinuria, and left ventricular hypertrophy

The correct answer is: Family history

119
Q

Which of the following types of cardiomyopathy presents with decreased left ventricular diastolic dimension and predominance of right-sided heart failure symptoms?

a. Dilated
b. Restrictive
c. Constrictive
d. Hypertrophic

A

The correct answer is: Restrictive

120
Q

Infiltration of abnormal substances between myocytes, storage of abnormal metabolic byproducts within myocytes or fibrotic injury are the usual causes of:

a. Dilated cardiomyopathy
b. Restrictive cardiomyopathy
c. Constrictive cardiomyopathy
d. Hypertrophic cardiomyopathy

A

The correct answer is: Restrictive cardiomyopathy

121
Q

What is the most infectious cause of cardiomyopathy worldwide?

a. HIV infection
b. Tuberculosis
c. Chagas’ disease
d. Coxsackie virus

A

The correct answer is: Chagas’ disease

122
Q

A 27 year old female developed fever, malaise, cough and nasal congestion 2 weeks ago. She does not have chest pain nor dyspnea. Her ECG showed diffuse T wave inversions in all leads. Troponin and CK-MB are elevated. Under what classification of myocarditis will this patient belong?

a. Possible subclinical acute myocarditis
b. Probable acute myocarditis
c. Definite myocarditis
d. Chronic myocarditis

A

The correct answer is: Possible subclinical acute myocarditis

123
Q

In a patients considered to have myocarditis, which diagnostic test with a positive result makes it a case of definite myocarditis?

a. 2D Echo
b. Cardiac MRI
c. Myocardial perfusion scintigraphy
d. Endomyocardial biopsy

A

The correct answer is: Endomyocardial biopsy

124
Q

Which of the following is TRUE of the most common infective cause of cardiomyopathy?

a. Histology reveals chronic granulomatous inflammation with Langhans type giant cells
b. It is transmitted by the bite of a reduviid bug
c. It is acquired from a bite of the tsetse fl
d. Its larva may invade the myocardium and is transmitted via ingestion of uncooked meat.

A

Histology reveals chronic granulomatous inflammation with Langhans type giant cells - TB
It is transmitted by the bite of a reduviid bug – Trypanosoma cruzi causing Chagas’ disease
It is acquired from a bite of the tsetse fly – Trypanosoma brucei causing African sleeping sickness
Its larva may invade the myocardium and is trasmitted via ingestion of uncooked meat. – Trichinella

The correct answer is: It is transmitted by the bite of a reduviid bug

125
Q

Peripartum cardiomyopathy develops during the last trimester of pregnancy and/or how many months after pregnancy?

a. 1
b. 2
c. 3
d. 6

A

The correct answer is: 6

126
Q

Which is the most common toxin implicated in chronic dilated cardiomyopathy?

a. Alcohol
b. Methamphetamine
c. Cocaine
d. Doxorubicin

A

The correct answer is: Alcohol

127
Q

The echocardiographic finding of global ventricular dilatation with basal contraction is typical of:

a. Duchenne’s muscular dystrophy
b. Lyme carditis
c. Diphtheritic myocarditis
d. Takotsubo cardiomyopathy

A

The correct answer is: Takotsubo cardiomyopathy

128
Q

Which medication is given to patients with hypertrophic cardiomyopathy as first line agent to alleviate symptoms of LV outflow tract obstruction?

a. Disopyramide
b. ACE inhibitor
c. Diuretic
d. Beta blocker

A

The correct answer is: Beta blocker

129
Q

Pulsus parvus et tardus, or the slow rise of the carotid pulse to a delayed peak, is a physical examination finding in which valvular disease?

a. Aortic stenosis
b. Pulmonic stenosis
c. Tricuspid stenosis
d. Mitral stenosis

A

The correct answer is: Aortic stenosis

130
Q

Aortic stenosis is said to be severe is the aortic valve area is less than:

a. 1.0 cm2
b. 1.5 cm2
c. 2.0 cm2
d. 2.5 cm2

A

The correct answer is: 1.0 cm2

131
Q

In patients with aortic stenosis, which manifestation portends the worst prognosis as patients usually die within 1.5 – 2 years of symptoms onset?

a. Angina pectoris
b. Syncope
c. Dyspnea
d. Congestive heart failure

A

The correct answer is: Congestive heart failure

132
Q

Which of the following is TRUE of the medical treatment of aortic stenosis?

a. Asymptomatic patients with severe AS can participate in strenuous physical activity and competitive sports.
b. Dehydration must be avoided.
c. All patients must be given infective endocarditis prophylaxis
d. Nitroglycerin is not helpful in relieving angina pectoris

A

The correct answer is: Dehydration must be avoided.

133
Q

What is the major hemodynamic compensation in aortic regurgitation?

a. Increased LV end-systolic volume
b. Increased LV end-diastolic volume
c. Decreased LV end-systolic volume
d. Decreased LV end-diastolic volume

A

The correct answer is: Increased LV end-diastolic volume

134
Q

What is the leading cause of mitral stenosis?

a. Congenital
b. Annular calcification
c. Infective endocarditis
d. Rheumatic fever

A

The correct answer is: Rheumatic fever

135
Q

Which chamber is enlarged in cases of mitral regurgitation but is of normal size in patients with mitral stenosis?

a. RA
b. RV
c. LA
d. LV

A

LV – the LV is not enlarged in MS because it is distal to the obstructed mitral valve. All the others are enarged bec they are proximal to the obstruction.

The correct answer is: LV

136
Q

Which of the following is NOT a consequence of severe mitral stenosis?

a. RV enlargement
b. LV enlargement
c. Pulmonary hypertension
d. Pulmonary regurgitation

A

All the rest are consequences of MS because of backward pressure
HPIM 20e, p. 1815

The correct answer is: LV enlargement

137
Q

Which of the following is a chest x-ray finding in patients with mitral stenosis?

a. Straightening of the upper right border of the cardiac silhouette
b. Diminution of the main pulmonary arteries
c. Constriction of the upper lobe pulmonary veins
d. Posterior displacement of the esophagus by an enlarged LA

A

The correct answer is: Posterior displacement of the esophagus by an enlarged LA

138
Q

Atrial septal defect may be mistaken for which valvular lesion since both conditions may have evidence of RV enlargement and accentuation of pulmonary vasculature?

a. Aortic stenosis
b. Mitral stenosis
c. Tricuspid regurgitation
d. Pulmonic regurgitation

A

The correct answer is: Mitral stenosis

139
Q

Which finding is seen in atrial septal defect?

a. RV enlargement
b. LA enlargement
c. Kerley B lines
d. Variable splitting of S2

A

RV enlargement is because of L to R shunt. There is no LV enlargement nor Kerley B lines.
There may be fixed splitting of S2.

The correct answer is: RV enlargement

140
Q

True about clinical manifestations of acute rheumatic fever:

a. The most commonly affected valve is the mitral valve, and the characteristic manifestation in previously affected individuals is mitral regurgitation
b. Joint involvement in ARF is usually symmetric and migratory, affecting large joints
c. Sydenham chorea can occur in isolation, without other features of rheumatic fever and are more common in females
d. Joint involvement often requires corticosteroids to control and suppress inflammation

A

The most commonly affected valve is the mitral valve, and the characteristic manifestation is mitral regurgitation. Joint involvement in ARF is usually asymmetric and migratory, highly responsive to steroids.
Sydenham chorea can occur in isolation, without other features of rheumatic fever and are more common in females

The correct answer is: Sydenham chorea can occur in isolation, without other features of rheumatic fever and are more common in females

141
Q

What is the indication of giving penicillin to patients with rheumatic mitral stenosis for what reason?

a. Prevention of infective endocarditis
b. Secondary prevention of rheumatic fever
c. Reduction in heart rate to allow diastolic filling
d. Prevention of thromboembolism

A

The correct answer is: Secondary prevention of rheumatic fever

142
Q

What is the optimal time to do cardioversion after patients with mitral stenosis and recent onset atrial fibrillation has achieved a therapeutic INR?

a. 3 days
b. 3 weeks
c. 30 days
d. 3 months

A

The correct answer is: 3 weeks

143
Q

Which of the ff best describes the murmur of mitral regurgitation?

a. early systolic and crescendo
b. mid systolic and plateau
c. late systolic and crescendo
d. holosystolic and plateau

A

The correct answer is: holosystolic and plateau

144
Q

What is the most common abnormality that lead to primary mitral regurgitation?

a. mitral valve prolapse
b. acute rheumatic fever
c. infective endocarditis
d. ruptured chordae tendineae

A

The correct answer is: mitral valve prolapse

145
Q

Which of the following is NOT part of the treatment of mitral valve prolapse?

a. Infective endocarditis prophylaxis in those with history of endocarditis
b. Beta blockers to relieve chest pain and palpitations
c. Anticoagulation in those with atrial fibrillation
d. Secondary rheumatic fever prophylaxis

A

The correct answer is: Secondary rheumatic fever prophylaxis

146
Q

Which of the following is a secondary cause of mitral regurgitations wherein the leaflets and chordae are “innocent bystanders?”

a. myxomatous
b. radiation
c. rheumatic fever
d. annular dilatation

A

The correct answer is: annular dilatation

147
Q

Which of the following is a chronic cause of mitral regurgitation?

a. infective endocarditis
b. papillary muscle rupture
c. chordal rupture
d. mitral valve prolapse

A

The correct answer is: mitral valve prolapse

148
Q

What is the most common cause of tricuspid stenosis?

a. congenital
b. rheumatic fever
c. infective endocarditis
d. carcinoid

A

The correct answer is: rheumatic fever

149
Q

In which valvular abnormality is the a wave extremely tall and the y descent prolonged?

a. mitral regurgitation
b. mitral stenosis
c. tricuspid regurgitation
d. tricuspid stenosis

A

The correct answer is: tricuspid stenosis

150
Q

Which of the following is TRUE of the signs and symptoms of tricuspid stenosis?

a. Edema, ascites and hepatomegaly are due to co-occurrence of mitral stenosis
b. Pulmonary congestion and fatigue are present when TS is severe
c. Jugular veins are characteristically flat.
d. The murmur of TS is increased during inspiration.

A

The correct answer is: The murmur of TS is increased during inspiration.

151
Q

The presence of tall peaked T waves and the absence of RV hypertrophy on ECG should prompt us to consider which valvular disease?

a. aortic stenosis
b. mitral stenosis
c. pulmonic stenosis
d. tricuspid stenosis

A

The correct answer is: tricuspid stenosis

152
Q

Pulsation of the liver during systole is a characteristic PE finding in which valvular abnormality?

a. tricuspid stenosis
b. tricuspid regurgitation
c. pulmonic stenosis
d. pulmonic regurgitation

A

The correct answer is: tricuspid regurgitation

153
Q

The most common cause of pulmonic stenosis is:

a. congenital
b. rheumatic
c. myxomatous
d. carcinoid

A

The correct answer is: congenital

154
Q

Which of the following is TRUE of the normal pericardium?

a. The visceral pericardium is a fibrous membrane while the parietal pericardium is a serous membrane.
b. Normal pericardial fluid volume is 150-500 mL
c. Pericardial fluid is an ultrafiltrate of plasma
d. Total absence of the pericardium, whether congenital or surgical, leads to deleterious effects.

A

Pericardial fluid is an ultrafiltrate of plasma
The visceral pericardium is a fibrous membrane while the parietal pericardium is a serous membrane – visceral is serous while parietal is fibrous. Normal pericardial fluid volume is 15 to 50 mL only. Total absence of the pericardium, whether congenital or surgical no deleterious effect

The correct answer is: Pericardial fluid is an ultrafiltrate of plasma

155
Q

Which is a characteristic feature of chest pain in acute pericarditis which may differentiate it from acute myocardial infarction?

a. retrosternal or left precordial in location
b. referred to the neck, arms or left shoulder
c. radiates to the trapezius ridge or into either arm
d. intensified by lying supine and relieved by leaning forward

A

The correct answer is: intensified by lying supine and relieved by leaning forward

156
Q

True statement about the clinical presentation of acute pericarditis:

a. Chest pain is pleuritic, and is relieved by lying supine or leaning forward
b. The classic pericardial friction rub is heard only in a minority of the patients
c. In larger pericardial effusions, the friction rub may be heard less
d. Widespread elevations of ST-segment elevations may occur, with downward concavity

A

Chest pain is pleuritic and is relieved by leaning forward and worsened by lying supine. The classic pericardial friction rub is heard in 85% of the patients. Widespread elevations of ST-segment elevations may occur, with upward concavity
In larger pericardial effusions, the friction rub may be heard less.

The correct answer is: In larger pericardial effusions, the friction rub may be heard less

157
Q

In acute pericarditis, what ECG finding suggests atrial involvement?

a. diffuse ST segment elevation
b. upwardly concave ST segment
c. Depression of the PR segment
d. Low voltage complexes

A

The correct answer is: Depression of the PR segment

158
Q

A 60-year old heavy smoker was admitted because of progressive easy fatigability that became dyspnea at rest in the past day. He also reported occasional hemoptysis in the past month. He appears diaphoretic, tachypneic and pale. BP is 110/60 that does not vary throughout the respiratory cycle, heart rate is 99 regular, RR is 25. His neck veins are engorged and rises during inspiration. Cardiac examination revealed displaced apex beat 6th ICS left anterior axillary line. What is the most likely diagnosis?

a. Cardiac tamponade
b. Constrictive pericarditis
c. Constrictive-effusive pericarditis
d. Acute pericarditis

A

The correct answer is: Constrictive pericarditis

159
Q

In patients with acute pericarditis, which of the following medicines is usually given for a period of 3 months, while the others should only be given for a few days to weeks?

a. anticoagulant
b. indomethacin
c. prednisone
d. colchicine

A

The correct answer is: prednisone

160
Q

Which of the following is NOT part of the Beck’s triad in cardiac tamponade?

a. Paradoxical pulse
b. Hypotension
c. Soft or absent heart sounds
d. jugular venous distention

A

The correct answer is: Paradoxical pulse

161
Q

In constrictive pericarditis, the inspiratory enlargement of the RV causes leftward bulging of the IV septum and reduced LV volume, stroke volume and arterial systolic pressure. These explain which finding?

a. electrical alternans
b. low voltage QRS complexes
c. pulsus paradoxus
d. pericardial friction rub

A

The correct answer is: pulsus paradoxus

162
Q

Grossly sanguineous pericardial fluid is not expected in which cause of pericardial effusion?

a. cancer
b. tuberculosis
c. renal failure
d. SLE

A

The correct answer is: SLE

163
Q

Which of the following is TRUE of the incidence of infective endocarditis (IE)?

a. Highest in the first 6-12 months after surgery for prosthetic valve
b. Bioprosthetic valves have higher risk for IE than mechanical prosthetic valves
c. Implantable cardioverter-defibrillators have the same risk as permanent pacemakers.
d. Incidence is higher in young adults compared to the elderly

A

The correct answer is: Highest in the first 6-12 months after surgery for prosthetic valve

164
Q

Injection drug use-associated infective endocarditis is usually due to what organism?

a. Viridans streptococci
b. Corynebacterium sp
c. Staphylococcus aureus
d. Haemophilus sp

A

The correct answer is: Staphylococcus aureus

165
Q

Injection drug use-associated infective endocarditis usually affects which cardiac valve?

a. mitral valve
b. tricuspid valve
c. aortic valve
d. pulmonic valve

A

The correct answer is: tricuspid valve

166
Q

Which of the following etiologies of infective endocarditis will manifest with high fever and rapidly progressing disease?

a. beta hemolytic streptococci
b. Bartonella sp.
c. Tropheryma whipplei
d. Coxiella burnetti

A

The correct answer is: beta hemolytic streptococci

167
Q

The most frequent symptom of infective endocarditis is:

a. Fever
b. Chills and sweats
c. Anorexia, weight loss, malaise
d. Myalgias, arthralgias

A

The correct answer is: Fever

168
Q

Which among the following is the most common sign of infective endocarditis?

a. heart murmur
b. arterial emboli
c. splenomegaly
d. petechiae

A

The correct answer is: heart murmur

169
Q

Which of the following is a major criterion in the modified Duke criteria for the clinical diagnosis of infective endocarditis?

a. HACEK group organism isolated from a single blood culture
b. fever >38.0 C
c. Increase in preexisting murmur
d. Coxiella burnetii phase I IgG antibody titer of >1:800

A

The correct answer is: Coxiella burnetii phase I IgG antibody titer of >1:800

170
Q

True about the etiology of infective endocarditis:

a. Endocarditis due to enterococci enter the bloodstream via the genitourinary tract
b. Injection drug use-associated endocarditis is usually caused by viridans streptococci
c. Health-care associated endocarditis is usually caused by gram-negative organisms
d. Up to 50% of endocarditis cases have negative blood cultures

A

Endocarditis due to enterococci enter the bloodstream via the genitourinary tract
Injection drug use-associated endocarditis is usually caused by S. aureus
Health-care associated endocarditis is usually caused by S. aureus, CONS, enterococci. Up to only 5-15% have culture-negative endocarditis.

The correct answer is: Endocarditis due to enterococci enter the bloodstream via the genitourinary tract

171
Q

According to the modified Duke criteria, which of the following manifestations of infective endocarditis is a vascular phenomenon and not an immunologic phenomenon?

a. Janeway lesions
b. Osler’s nodes
c. Roth’s spots
d. glomerulonephritis

A

The correct answer is: Janeway lesions

172
Q

Which of the following test is nondiagnostic but important to be performed in the management of infective endocarditis?

a. electrocardiogram
b. blood culture
c. echocardiogram
d. PCR of microbial DNA

A

The correct answer is: electrocardiogram

173
Q

Which of the following conditions will warrant emergent (same day) surgery for infective endocarditis?

a. Fungal endocarditis
b. Septal perforation
c. Valve dysfunction with pulmonary edema or cardiogenic shock
d. Valve dysfunction plus persisting infection after >7-10 days of antimicrobial infection

A

The correct answer is: Valve dysfunction with pulmonary edema or cardiogenic shock

174
Q

In which of the following procedures is infective endocarditis prophylaxis indicated among patients with high-risk cardiac lesions?

a. Manipulation of gingival tissue
b. Upper GI endoscopy
c. Colonoscopy
d. Cystoscopy

A

The correct answer is: Manipulation of gingival tissue

175
Q

A 25 year old male known to have RHD has been having low grade fever for the past 3 weeks. He now presents with fatigue and dyspnea. Which empiric therapy will you start if you are considering infective endocarditis?

a. Oxacillin + gentamicin
b. Vancomycin + ceftriaxone
c. Penicillin + gentamicin
d. Ampisulbactam + doxycycline

A

The correct answer is: Vancomycin + ceftriaxone

176
Q

What is the standard oral regimen for infective endocarditis prophylaxis involving amoxicillin?

a. 500 mg x 1 capsule, single dose 1 hour before procedure
b. 500 mg x 4 capsules, single dose 1 hour before procedure
c. 500 mg TID starting 1 day before the procedure
d. 500 mg TID starting 7 days before the procedure

A

The correct answer is: 500 mg x 4 capsules, single dose 1 hour before procedure